LSAT and Law School Admissions Forum

Get expert LSAT preparation and law school admissions advice from PowerScore Test Preparation.

 Adam Tyson
PowerScore Staff
  • PowerScore Staff
  • Posts: 5153
  • Joined: Apr 14, 2011
|
#48178
While it worked for you this time, khodi, I would say that was a risky way to approach these answer choices, because it would have been very easy to craft a bad answer that did mention the overall population and a correct one that did not. For example:
People with university educations are only half as likely to make use of government social services than are members of the overall population.
Completely irrelevant to the issue of who may or may not support reducing the level of those services, so a wrong answer.
People with a university education tend to maintain their belief systems from the time they graduate until the end of their lives.
If these people never change their minds again, that would strengthen the claim that a higher percentage of them believe in retaining or increasing social services than does the overall population, even though we never mentioned the overall population.

Mechanical approaches can be very useful, but we have to be careful to avoid over-simplifying them and losing sight of the context and meaning of each stimulus, stem, and answer choice. While a mention of the overall population might be enough to call an answer a contender, its absence isn't enough here to make it a loser, nor is its presence sufficient to make it the best answer.
 jayzbrisk
  • Posts: 12
  • Joined: Mar 19, 2019
|
#65562
I had a problem with answer choice D because the argument says "These polls lead us...." doesnt that mean that the argument is based on these polls alone, independent of the polls of choice D, even if when added together it weakens the argument?
 George George
PowerScore Staff
  • PowerScore Staff
  • Posts: 48
  • Joined: Jun 07, 2019
|
#65618
@jayzbrisk

This is actually a great Q! There are two things to address.

First, the phrase "These polls lead us to the conclusion that..." is a Conclusion Indicator. It tells you that the conclusion follows. So the conclusion is really the clause "People with a university education are more likely to favor retaining or increasing the present level of government social services than are members of the overall population." However, it is also fair to say that this conclusion is only based on those polls of university students. (The first sentence is the only premise in this argument.)

Second, however, just because this conclusion is only based on a few polls does not mean that this conclusion was about those polls. And, third, in a Weakener Q generally, the answers almost always bring in New Info like other polls or studies, as in (D). This is always permitted on a Weakener Q, and the issue for you is what impact that new information would have on the argument's conclusion. So, here, (D) is relevant because it would undercut the idea that university educated people prefer government social services more than the general public.
 AlyssaY
  • Posts: 14
  • Joined: Sep 30, 2019
|
#80528
Dave Killoran wrote: Answer choice (E): This can be an attractive answer at first because the strength of opinion factor suggests to some people that the polling didn't completely capture attitudes and opinions fully or correctly. But the argument never references the concept of strength, just the general opinion, and we do know what those opinions were. Thus, for this answer to work, we'd need a lot more information otherwise you are making assumptions.
If answer E had said something like "In the polls cited, graduating university students were more likely to answer questions on reducing government social services than were students entering a university" instead of "express strong opinions about the question" -- would that be enough to weaken the argument since it calls into question how reliably the stats reflect the opinions of graduating versus first year students?

I can now see why D is the correct answer but I initially chose E and I'm curious for other weakening questions moving forward. Thank you so much!
User avatar
 KelseyWoods
PowerScore Staff
  • PowerScore Staff
  • Posts: 1079
  • Joined: Jun 26, 2013
|
#81253
Hi Alyssa!

Changing answer choice (E) in that way still wouldn't make it a great answer choice here. It might slightly call into question the reliability of the stats, but you'd still be making some pretty big assumptions about what being more or less likely to answer those questions actually means. Is it just a reflection of the strength of their opinions? If so, that's already what answer choice (E) says and it doesn't attack the argument. Is it that the survey glitched more for first year students than graduating students and just happened to skip those questions? If so, that still doesn't necessarily mean that the data from the two groups isn't comparable (here you'd have to get into the statistical power of your sample sizes and more advanced statistics concepts that the LSAT is not going to test you on). So, again, changing answer choice (E) in that way is not really going to weaken the argument and would certainly not be a better answer choice than (D).

Hope this helps!

Best,
Kelsey

Get the most out of your LSAT Prep Plus subscription.

Analyze and track your performance with our Testing and Analytics Package.